wj097 Wrote:I have 2 interpretation as to why (E) is correct, but not sure which one is the proper approach.
1. It strengthens the conclusion on an separate independent base regardless of the premise.
2. It strengthens the argument by defending from possible situation that undermines the necessary assumption ("other factor has not changed significantly to outweigh the continued profitable TV ad selling environment")
Thx
It's rare that a strengthen/weaken answer will go after the conclusion instead of the argument. I see this fitting into the majority. The argument's assumption, as you reference, is that there isn't some other factor that makes the continued profits outweighed by the rising costs. (E) tells us that indeed, the outweighing has not happened.
Thinking aloud here: does (E) really support the conclusion very well? I don't think so. Just because viewership is going up can we say it'll be no harder to sell ads next fall? That's a pretty weak argument since there are so many reasons ads could be harder to sell regardless of rising viewership.